Holomorphic function convergent sequence

Click For Summary
The discussion focuses on a homework problem related to holomorphic functions and their properties concerning convergence and zeros. The user is seeking clarification on the origin of the inequality |g(z)| ≥ |a_m|/2, particularly in relation to the first non-zero Fourier coefficient a_m and its implications for higher terms. Additionally, there is confusion about the conclusion that f(z) has only one zero at z=z_0, despite knowing that g(z_0) = a_m ≠ 0 and a_0 = 0. The user expresses uncertainty about the interpretation of a lemma regarding the conditions under which f(z_n) = 0, highlighting the potential for differing outcomes based on the interpretation. Overall, the thread emphasizes the need for clarity in mathematical statements and theorems related to holomorphic functions.
binbagsss
Messages
1,291
Reaction score
12

Homework Statement


[/B]
Hi

Theorem attached and proof.

canidieyet.png


I am stuck on

1) Where we get ##|g(z)|\geq |a_m|/2 ## comes from
so ##a_{m}## is the first non-zero Fourier coeffient. So I think this term is ##< |a_m|r^{m}##, from ##r## the radius of the open set, but I don't know how to take care of the rest of the higher tems through ##a_{m}## , is this some theorem or?

2) The conclusion thus ##f(z)## has only one zero at ##z=z_0##
I think I'm being stupid but what is this being made from?
We know ##g(z_0) = a_{m} \neq 0 ## and ##a_{0}=0##, but I don't understand.

Thanks

Homework Equations


above

The Attempt at a Solution


above
 
Physics news on Phys.org
The Lemma is very badly expressed. What do you think it means by 'if ##f(z_n)=0## for any ##n##, then...'? This vague phrase could either mean
(1) ''if there exists some ##n\in\mathbb N## such that ##f(z_n)=0##, then..."
or it could mean
(2) "if for every natural number ##n##, ##f(z_n)=0##, then ..."

The two interpretations have very different consequences.
 
Question: A clock's minute hand has length 4 and its hour hand has length 3. What is the distance between the tips at the moment when it is increasing most rapidly?(Putnam Exam Question) Answer: Making assumption that both the hands moves at constant angular velocities, the answer is ## \sqrt{7} .## But don't you think this assumption is somewhat doubtful and wrong?

Similar threads

  • · Replies 1 ·
Replies
1
Views
1K
  • · Replies 2 ·
Replies
2
Views
2K
Replies
8
Views
2K
Replies
3
Views
2K
  • · Replies 4 ·
Replies
4
Views
2K
  • · Replies 7 ·
Replies
7
Views
2K
  • · Replies 1 ·
Replies
1
Views
2K
  • · Replies 2 ·
Replies
2
Views
2K
  • · Replies 1 ·
Replies
1
Views
1K
  • · Replies 9 ·
Replies
9
Views
2K